Soal Otl
Soal Otl
Soal Otl
1. Figures above are the clinical photograph and radiographs of a 5-year old boy who fell
and injured his right elbow. His radial pulse is thready. Which neurologic deficit most
commonly is associated with this injury?
Answer: B
Reference: Anatomy, Shoulder and Upper Limb, Radial Nerve - PubMed (nih.gov)
2. Figures above are the radiographs of an 11-year-old boy with a prolonged history of
activity-related ankle pain. An examination is notable for restricted subtalar motion and
moderate pes planovalgu. The diagnosis is:
A. Frelberg infarction
B. Tarsal coalition
C. Accessory navicular
D. Neglected clubfoot
Answer: B
Tarsal Coalition is a common congenital condition caused by failure of embryonic
segmentation leading to abnormal coalition 2 or more of the tarsal bones. The condition is
usually asymptomatic, but may present with a flatfoot deformity or recurrent ankle
sprains.
a. Diagnosis is made with plain radiographs of the foot and ankle showing a coalition,
most commonly a calcaneonavicular or talocalcaneous coalition.
b. Treatment is usually a course of casting and NSAIDs for symptomatic patients.
Surgical coalition resection or joint arthrodesis is indicated for patients with
persistent symptoms who fail conservative management.
Reference: Tarsal Coalition - Pediatrics - Orthobullets
3. Which inheritance pattern has been described for the condition shown in Figures above?
A. Autosomal recessive
B. Autosomal dominant.
C. X-linked recessive
D. X-linked dominant
Answer: B
Two types
a. Congenital
Most common
b. Acquired
Less common and caused by
1. Trauma
2. Degenerative
3. Infections
Associated conditions
c. Nonsyndromic
Autosomal Dominant
d. Syndromic
Fibular hemimelia
Carpal coalition
FGFR-associated craniosynostosis (FGFR-1, FGFR-2, FGFR-3)
Apert syndrome, Pfeiffer, Crouzon, Jackson-Weiss and Muenke
Reference: Tarsal Coalition - Pediatrics - Orthobullets
Answer: B
Syndromic
Fibular hemimelia
Carpal coalition
FGFR-associated craniosynostosis (FGFR-1, FGFR-2, FGFR-3)
Apert syndrome, Pfeiffer, Crouzon, Jackson-Weiss and Muenke
Reference: Tarsal Coalition - Pediatrics - Orthobullets
5. Figures above are the radiographs of an 11-year old boy with a prolonged history of
activity-related ankle pain. An examination is notable for restricted subtalar motion and
moderate pes planovalgus. The patient fails nonsurgical treatment.
What is the best next step?
A. CT-scan|
B. MRI
C. Bone scan
D. laboratory studies: complete blood count (CBC) with differential, erythrocyte
sedimentation rate (ESR), CRP, antinuclear antibody (ANA), and rheumatoid factor
(RF)
Answer: B
Given the patient’s symptoms, restricted subtalar motion, and pes planovalgus, the best
next step would be to obtain an MRI (Magnetic Resonance Imaging) of the affected
ankle.
1. MRI:
○ MRI provides excellent soft tissue visualization, allowing assessment of
ligaments, tendons, cartilage, and bone marrow.
○ It can identify subtle abnormalities such as osteochondral lesions, ligament
injuries, and inflammation.
○ In this case, MRI can help evaluate the extent of joint involvement, assess for
osteonecrosis (as seen in Freiberg’s disease), and detect any other soft tissue
pathology.
Reference: Tarsal Coalition - Pediatrics - Orthobullets
6. The CT scan shows the involvement area is approximately 30% of posterior facet. What
is the most appropriate treatment?
A. Surgical resection
B. Lateral column lengthening.
C. Coallition resection and lateral column lengthening|
D. Triple arthrodesis.
Answer: C
Given the involvement area of approximately 30% of the posterior facet, let’s consider
the treatment options:
1. Surgical Resection:
○ Surgical resection involves removing the coalition (abnormal bony connection)
between the tarsal bones.
○ It is a reasonable option for smaller coalitions or when the involvement is limited.
○ However, in cases with a larger involvement area, resection alone may not be
sufficient.
2. Lateral Column Lengthening:
○ Lateral column lengthening (Evans osteotomy) aims to correct pes
planovalgus by lengthening the lateral column of the foot.
○ It can be effective in improving foot alignment and reducing symptoms.
○ However, it does not directly address the coalition itself.
3. Coalition Resection and Lateral Column Lengthening:
○ This combined approach involves both resecting the coalition and performing
lateral column lengthening.
○ It addresses both the coalition and the associated deformity.
○ It is often preferred for larger coalitions or cases with significant pes planovalgus.
4. Triple Arthrodesis:
○ Triple arthrodesis is a more extensive procedure that fuses the subtalar,
talonavicular, and calcaneocuboid joints.
○ It is typically reserved for severe cases of pes planovalgus and coalition.
○ While effective, it results in loss of motion in the fused joints.
Recommendation: Considering the involvement area and the desire to address both
the coalition and pes planovalgus, the coalition resection and lateral column
lengthening approach would likely be the most appropriate treatment. However, this
decision should be made in consultation with an orthopedic surgeon who can assess
the patient’s specific condition and discuss the pros and cons of each option.
7. The pathogenesis of this condition is associated with
A. A PITX1 mutation.
B. A point polymorphism in a regulatory sequence for the COLIA-1 gene.
C. Mutations in the EXT gene family.
D. Mutations in the COL5A or COLA genes.
Answer: C
The pathogenesis of this condition is associated with mutations in the EXT gene family.
These mutations are specifically related to tarsal coalition, a condition where abnormal
bony connections form between tarsal bones in the foot. The EXT gene family plays a
role in skeletal development, and alterations in these genes can lead to abnormal bone
formation and coalition.
Answer: B
The idiopathic form of this condition is associated with absence of the fibula. Fibular
hemimelia, characterized by partial or complete absence of the fibula, can lead to various
foot and ankle anomalies, including tarsal coalition. The absence of the fibula disrupts
normal skeletal development and can contribute to coalition formation.
Answer: B
The bony abnormalities in this condition occur mostly in the tarsal bones. Tarsal coalition
involves abnormal connections between two or more tarsal bones, leading to restricted
motion and potential pain. The most common sites for coalition are the talocalcaneal
(between the talus and calcaneus) and calcaneonavicular (between the calcaneus and
navicular) joints. These bony abnormalities can impact foot function and cause
symptoms.
Reference: Clubfoot (congenital talipes equinovarus) - Pediatrics - Orthobullets
10. The infant underwent Ponseti casting for 5 weeks. Afterward, a heel cord release was
done 1 cm proximal to the insertion site of the Achilles tendon and was casted in long-leg
casts with the knee flexed and an external rotation mold on the leg for 3 weeks. Straight-
last shoes with an abduction bar set at 70 degrees of external rotation were fitted. The
infant's feet remained in the corrected position at a 1-month check, but, at a 3-month
check, ankle equinus and forefoot varus were present.
Recurrence of the deformity is most likely attributable to
A. Noncompliance with postsurgical bracing
B. Insufficient length of time in postsurgical casts.
C. Proximal placement of the incision for heel cord release.
D. Excessive rotation of postsurgical bracing.
Answer: C
The recurrence of the deformity after Ponseti casting and heel cord release can be
influenced by several factors. Let’s evaluate the options:
1. Noncompliance with Postsurgical Bracing:
○ Compliance with the prescribed orthotic bracing is crucial for maintaining
correction after clubfoot treatment.
○ If the parents or caregivers did not consistently follow the recommended bracing
schedule, it could contribute to recurrence.
2. Insufficient Length of Time in Postsurgical Casts:
○ The initial casting phase (6 to 8 weeks) is essential for achieving complete
correction of the clubfoot deformity.
○ If the casting duration was inadequate, the foot may not have fully reshaped,
leading to recurrence.
3. Proximal Placement of the Incision for Heel Cord Release:
○ The heel cord release (Achilles tendon lengthening) is performed to address
equinus (ankle plantarflexion).
○ If the incision was placed too proximal (closer to the Achilles insertion), it might
not have effectively released the tight tendon, resulting in persistent equinus.
4. Excessive Rotation of Postsurgical Bracing:
○ The abduction bar set at 70 degrees of external rotation is crucial for maintaining
the corrected foot position.
○ If the external rotation was excessive or insufficient, it could impact the alignment
and lead to recurrence.
Reference: Clubfoot (congenital talipes equinovarus) - Pediatrics - Orthobullets
11. A 58 year-old man has had increasing midback pain for 8 weeks. Radiographs 1 poin
reveal mild osteopenia and mild disk degeneration but no fracture or lesions An MRI of
the spine reveals diskitis with a small-intensity signal within the spinal canal that is
consistent with an epidural abcess at T11-12. The patient is neurologically intact but in
signifanct pain. CT-guided biopsy of the disk space is positive for methicillin-sensitive
staphylococcus aureus. What is the most appropriate treatment?
A. Intervenous (IV) antibiotics for 6 weeks and clinical observation.
B. Observation and bracing alone
C. Laminectomy and posterior spinal fusion with IV antibiotics
D. Anterior spinal debridement and fusion with IV antibiotics
Answer: C
Reference: https://www.ncbi.nlm.nih.gov/books/NBK441890/
12. A 60-years-old woman has severe neck and back pain. She is relatively healthy but has
diabetes and neuropathy involving her lower extremities. Her body mass indeks is 38. She
has a history of spinal fusion performed by your colleague 3 years ago. At that time, she
was treated for degenerative scoliosis of lumbar spine with concomitant spinal stenosis.
360-degree fusion was performed from L4-S1 with a posterior decompression from L2-
S1 and a posterior instrumented fusion from T3 to pelvis. On examination, she has
reproducible pain and visible kyphosis in the periscapular region Neurologic examination
findings are within normal limits, with the exception of lower-extremity dysesthesias
related to her neuropathy. The patient states that she has readiographs and a standing
scoliosis series demonstrate a solid fusion from the sacrum to T3 without evidence of
hardware failure. There is focal collapse of the T2-T3 disk space and proximal kyphosis
involving the T2 vertebrae that is indicative of disk and ligamentous failure. The patient
ask why the proximal kyphosis occured. You indicate that she has several risk factors for
this condition, including
A. Her Age, Fusion to the sacrum, and Upper instrumented vertebra at T3
B. Her age, 360 degree fusion, and obesity
C. Diabetes, history of neuropathy, and pelvic instrumentation
D. Upper instrumentation vertebra at T3, obesity, and diabetes
Answer: A
The proximal kyphosis observed in this patient can be attributed to several risk factors.
Let’s explore them:
1. Age:
○ Advanced age is a significant risk factor for the development of proximal
junctional kyphosis (PJK).
○ As individuals age, changes in bone density, ligamentous laxity, and overall spinal
biomechanics can contribute to kyphotic deformities.
2. Fusion to the Sacrum:
○ The fusion extending to the sacrum (L4-S1) is associated with an increased risk of
PJK.
○ The transition between the rigidly fused lumbar spine and the more mobile sacrum
can create stress concentration at the adjacent levels.
3. Upper Instrumented Vertebra at T3:
○ The choice of the upper instrumented vertebra (UIV) plays a crucial role.
○ In this case, the UIV was at T3, which is relatively high in the thoracic spine.
○ High UIVs are associated with a higher risk of PJK due to increased
biomechanical stress at the adjacent levels.
Considering these risk factors, it’s essential to monitor patients who undergo spinal
fusion for signs of PJK. Collaborating with a spine specialist and considering
preventive measures can help manage and minimize the risk of complications.
Refence: Risk factors for proximal junctional kyphosis after posterior long-segment
internal fixation for chronic symptomatic osteoporotic thoracolumbar fractures with
kyphosis | BMC Surgery | Full Text (biomedcentral.com)
13. A 60-years-old woman has severe neck and back pain. She is relatively healthy but has
diabetes and neuropathy involving her lower extremities. Her body mass index is 38. She
has a history of spinal fusion performed by your colleague 3 years ago. At that time, she
was treated for degenerative scoliosis of lumbar spine with concomitant spinal stenosis.
36-degree fusion was performed from L4-S1 with a posterior decompression from L2-S1
and a posterior instrumented fusion from T3 to pelvis. On examination, she has
reproducible pain and visible kyphosis in the periscapular region. Neurologic examination
findings are within normal limits, with the exception of lower-extremity dysesthesias
related to her neuropathy. The patient states that she has radiographs and a standing
scoliosis series demonstrating a solid fusion from the sacrum to T3 without evidence of
hardware failure. There is focal collapse of the T2-T3 disk space and proximal kyphosis
involving the T2 vertebrae that is indicative of disk and ligamentous failure.
Considering her current symptoms, history, and examination findings, you believe that
the most efficacious means with which to address this patient's junctional kyphosis is
A. Cervico-thoracic- lumbosacral orthosis to immobilize the kyphosed segment
B. Removal of the posterior instrumentation because she has had a successful fusion
C. Anteroposterior instrumented fusion at T2-3 to address the collapsed interspace
D. Revision Posterior Instrumentation with Extension of the Fusion to T1 to Span the
area of junctional kyphosis
Answer: D
Given the patient’s history of spinal fusion, visible kyphosis, and focal collapse of the T2-
T3 disk space, addressing the proximal junctional kyphosis (PJK) is crucial. Let’s
evaluate the options:
1. Cervico-Thoracic-Lumbosacral Orthosis (CTLSO):
○ CTLSO is an external brace that provides support and immobilization.
○ While it can help stabilize the spine, it may not directly address the collapsed
interspace or ligamentous failure.
2. Removal of Posterior Instrumentation:
○ Removing the posterior instrumentation is unlikely to be the best approach.
○ The patient has a solid fusion from the sacrum to T3, and hardware removal may
compromise stability.
3. Anteroposterior Instrumented Fusion at T2-3:
○ Anteroposterior fusion at T2-3 would directly address the collapsed interspace.
○ However, it may not fully prevent further kyphosis or address the underlying
ligamentous failure.
4. Revision Posterior Instrumentation with Extension to T1:
○ Extending the fusion to T1 would span the area of junctional kyphosis.
○ This approach aims to stabilize the adjacent levels and prevent further deformity.
Recommendation: Considering the patient’s symptoms and history, the most efficacious
means to address the junctional kyphosis is revision posterior instrumentation with
extension of the fusion to T1. This approach provides comprehensive stabilization and
addresses both the collapsed interspace and ligamentous failure. However, the final
decision should be made in consultation with the treating spine surgeon based on
individualized assessment and surgical planning.
Reference: https://bmcsurg.biomedcentral.com/articles/10.1186/s12893-022-01598-9
14. A 60-years-old woman has severe neck and back pain. She is relatively healthy but has
diabetes and neuropathy involving her lower extremities. Her body mass indeks is 38. She
has a history of spinal fusion performed by your colleague 3 years ago. At that time, she
was treated for degenerative scoliosis of lumbar spine with concomitant spinal stenosis.
360-degree fusion was performed from L4-$1 with a posterior decompression from L2-$1
and a posterior instrumented fusion from T3 to pelvis. On examination, she has
reproducible pain and visible kyphosis in the periscapular region. Neurologic examination
findings are within normal limits, with the exception of lower-extremity dysesthesias
related to her neuropathy. The patient states that she has readiographs and a standing
scoliosis series demonstrate a solid fusion from the sacrum to T3 without evidence of
hardware failure. There is focal collapse of the T2-T3 disk space and proximal kyphosis
involving the T2 vertebrae that is indicative of disk and ligamentous failure. What are this
patient's risk factors for postsurgical infection?
A. Presence of posterior spinal instrumentation
B. Anterior approach performed during the indeks procedure
C. Diabetes and Obesity
D. Surgery performed in the thoracic spine
Answer: C
Potential risk factors for postsurgical infection after spinal fusion:
1. Obesity:
○ Obesity is a significant risk factor for surgical site infection (SSI) following spinal
fusion.
○ Patients with a higher body mass index (BMI) (such as your patient with a BMI of
38) are more susceptible to infections1.
2. Diabetes:
○ Patients with diabetes have impaired wound healing and an increased risk of
infections.
○ Poor glycemic control can further exacerbate this risk1.
3. Smoking Status:
○ Tobacco use is associated with a higher incidence of SSI.
○ Smoking negatively affects tissue oxygenation and immune response, making
patients more vulnerable to infections1.
4. Operative Time:
○ Longer surgical procedures are linked to an increased risk of infection.
○ Prolonged exposure during surgery provides more opportunities for
contamination1.
5. Revision Surgery:
○ Patients who have undergone previous spinal surgery (such as your patient’s
history of spinal fusion) are at higher risk for infections.
○ Revision surgeries carry additional risks due to altered tissue planes and scar
tissue1.
6. Extension of Fusion to the Sacrum or Pelvis:
○ Extending the fusion to the sacrum or pelvis increases the surgical complexity.
○ More extensive fusion areas may lead to a higher risk of infection1.
7. Other Factors:
○ Although not directly modifiable, factors such as the American Society of
Anesthesiologists (ASA) score (reflecting overall health status) and age also play
a role in infection risk1.
Reference: What are the risk factors for surgical site infection after spinal fusion? A meta-
analysis | European Spine Journal (springer.com)
15. A 60-years-old woman has severe neck and back pain. She is relatively healthy but has
diabetes and neuropathy involving her lower extremities. Her body mass indeks is 38. She
has a history of spinal fusion performed by your colleague 3 years ago. At that time, she
was treated for degenerative scoliosis of lumbar spine with concomitant spinal stenosis.
360-degree fusion was performed from L4-$1 with a posterior decompression from L2-$1
and a posterior instrumented fusion from T3 to pelvis. On examination, she has
reproducible pain and visible kyphosis in the periscapular region. Neurologic examination
findings are within normal limits, with the exception of lower-extremity dysesthesias
related to her neuropathy. The patient states that she has readiographs and a standing
scoliosis series demonstrate a solid fusion from the sacrum to T3 without evidence of
hardware failure. There is focal collapse of the T2-T3 disk space and proximal kyphosis
involving the T2 vertebrae that is indicative of disk and ligamentous failure. Which
complication is most frequently encountered after revision surgery to treat junctional
kyphosis?
A. Deep surgical site infection
B. Pseudoarthrosis
C. Neurological injury
D. adjacenet segment degeneration
Answer: D
The most frequently encountered complication after revision surgery to treat proximal
junctional kyphosis (PJK) is adjacent segment degeneration. Let me explain why:
Recommendation:
Answer: C
The reading of 2 mA during triggered electromyography (EMG) testing of the right T2
pedicle screw indicates that there is a breach in the right T2 pedicle wall, but the screw is
not in contact with a neural structure. This finding suggests that the pedicle screw
placement may be suboptimal, and further evaluation and adjustment are necessary to
ensure proper positioning and prevent neural injury
Reference: https://bmcsurg.biomedcentral.com/articles/10.1186/s12893-022-01598-9
17. A 60-years-old woman has severe neck and back pain. She is relatively healthy but has
diabetes and neuropathy involving her lower extremities. Her body mass indeks is 38. She
has a history of spinal fusion performed by your colleague 3 years ago. At that time, she
was treated for degenerative scoliosis of lumbar spine with concomitant spinal stenosis.
360-degree fusion was performed from L4-S1 with a posterior decompression from L2-
S1 and a posterior instrumented fusion from T3 to pelvis. On examination, she has
reproducible pain and visible kyphosis in the periscapular region Neurologic examination
findings are within normal limits, with the exception of lower-extremity dysesthesias
related to her neuropathy. The patient states that she has readiographs and a standing
scoliosis series demonstrate a solid fusion from the sacrum to T3 without evidence of
hardware failure. There is focal collapse of the T2-T3 disk space and proximal kyphosis
involving the T2 vertebrae that is indicative of disk and ligamentous failure. What is the
most common adverse postoperative complication of laminoplasty for multilevel cervical
spondylotic myelopathy?
A. Loss of cervical range of motion
B. Inadvertent closure of the laminoplasty postoperatively
C. Progressive cervical kyphosis
D. C5 nerve root palsy
E. Inadequate decompression
Answer: D
Reference: https://www.ncbi.nlm.nih.gov/pmc/articles/PMC5400195/
18. A 60-years-old woman has severe neck and back pain. She is relatively healthy but has
diabetes and neuropathy involving her lower extremities. Her body mass indeks is 38. She
has a history of spinal fusion performed by your colleague 3 years ago. At that time, she
was treated for degenerative scoliosis of lumbar spine with concomitant spinal stenosis.
360-degree fusion was performed from L4-S1 with a posterior decompression from L2-
S1 and a posterior instrumented fusion from T3 to pelvis. On examination, she has
reproducible pain and visible kyphosis in the periscapular region Neurologic examination
findings are within normal limits, with the exception of lower-extremity dysesthesias
related to her neuropathy. The patient states that she has readiographs and a standing
scoliosis series demonstrate a solid fusion from the sacrum to T3 without evidence of
hardware failure. There is focal collapse of the T2-T3 disk space and proximal kyphosis
involving the T2 vertebrae that is indicative of disk and ligamentous failure. In a
retroperitoneal approach to the lumbar spine, what structure runs along the medial aspect
of the psoas and long the lateral border of the spine?
A. Illioinguinal nerve
B. Genitofemoral nerve
C. Sympathetic trunk.
D. Ureter
E. Aorta
Answer: D
In a retroperitoneal approach to the lumbar spine, the structure that runs along the medial
aspect of the psoas muscle and extends along the lateral border of the spine is the ureter.
The ureter lies between the psoas fascia and the peritoneum and is more firmly attached
to the peritoneum. During surgery, it’s crucial to be aware of the ureter’s location to avoid
inadvertent injury.
Reference: Retroperitoneal (Anterolateral) Approach to the Lumbar Spine - Approaches -
Orthobullets
19. A previously healthy 30-year-old woman has neck pain and bilateral hand and 1 poin
lower extremity tingling with weakness after falling down stairs. She is alert and oriented.
Examination reveals incomplete quadriplegia at the C6 level that remains unchanged
throughout her evaluation and initial treatment. Radiographs show a bilateral facet
dislocation of C6 on C7 without fracture. Attempts ar reduction with halo cervical
traction up to her body weight are unsuccesful. What is the most appropriate next step?
A. Posterior open reduction and fusion with fixation.
B. Anterior open reduction and fusion with fixation.
C. Technetiucm TC-99m bone scan.
D. Closed manipulation.
E. MRI
Answer: A
In this case, the most appropriate next step for the patient with bilateral facet dislocation
of C6 on C7 without fracture is surgical stabilization. Let’s break down the reasoning:
20. A previously healthy 30-year-old woman has neck pain and bilateral hand and 1 poin
lower extremity tingling with weakness after falling down stairs. She is alert and oriented.
Examination reveals incomplete quadriplegia at the C6 level that remains unchanged
throughout her evaluation and initial treatment. Radiographs show a bilateral facet
dislocation of C6 on C7 without fracture. Attempts are reduction with halo cervical
traction up to her body weight are unsuccesful. When harvesting iliac crest bone graft
during a posterior spinal decompression and fusion, injury to what structure can result in
painful neuromas or numbness over the skin of the buttocks?
A. Illioinguinal nerve
B. Superior Gluteal nerve.
C. Superior Cluneal Nerves
D. Illiohypogastric nerves
E. Lateral Femoral Cutaneous nerve
Answer: D
21. A 45-year-old woman with diabetes has a 3-month history of left shoulder pain and
motion loss unrelated to trauma. She previously underwent treatment with nonsteroidal
anti-inflammatory medication and a home stretching program, experiencing minimal
relief of her symptoms. Examination reveals loss of passive external rotation, abduction,
and forward elevation without reduction in strength.
Radiograph findings are normal. What is the most appropriate next step?
A. MRI scan
B. Cortisone injection therapy with continued physical therapy (PT)
C. Closed manipulation under anesthesia
D. Arthroscopic release with manipulation under anesthesia
Answer: B
22. A 42-year-old woman has a posterior elbow dislocation. Closed reduction is performed,
and the elbow appears stable under fluoroscopic examination. Further treatment should
consist of
A. Early mobilization only
B. Surgical repair or reconstruction of the lateral collateral ligament (LL) and the medial
collateral ligament (MCL).
C. Active range of motion in a hinged brace with a range of 30 degrees to 120 degrees.
D. Application of a hinged external fixator with early mobilization
Answer: C
Reference: Elbow Dislocation | Concise Medical Knowledge (lecturio.com)
23. A 45-year-old woman with diabetes has a 3-month history of left shoulder point pain and
motion loss unrelated to trauma. She previously underwent treatment with nonsteroidal
anti-inflammatory medication and a home stretching program, experiencing minimal
relief of her symptoms. Examination reveals loss of passive external rotation, abduction,
and forward elevation without reduction in strength.
Radiograph findings are normal. What is the most appropriate next step?
A. MRI scan.
B. Cortisone injection therapy with continued physical therapy (PT)
C. Closed manipulation under anesthesia.
D. Arthroscopic release with manipulation under anesthesia
Answer: B
Sumber: Clinical Guidelines in the Management of Frozen Shoulder: An Update! - PMC
(nih.gov)
24. A 42-year-old woman has a posterior elbow dislocation. Closed reduction point is
performed, and the elbow appears stable under fluoroscopic examination.
Further treatment should consist of
A. Early mobilization only.
B. Surgical repair or reconstruction of the lateral collateral ligament (LL) and the medial
collateral ligament (MCL).
C. Active range of motion in a hinged brace with a range of 30 degrees to 120 degrees.
D. Application of a hinged external fixator with early mobilization.
Answer: C
Reference: Elbow Dislocation | Concise Medical Knowledge (lecturio.com)
25. 36-year-old right-hand-dominant man fell from his motorcycle and sustained the acute
right upper extremity injury seen in Figure 18. At surgery, an open reduction and internal
fixation of the ulna is performed along with attempted open reduction of the radio-
capitellar joint. However the radial head is slightly subluxed in flexion and re-dislocates
with elbow extension below 90 degrees. What is the most appropriate treatment at this
time?
A. Radial head resection
B. Casting in 90 degrees of flexion for 3 weeks, followed by reassessment of elbow
stability
C. Repair of the annular ligament
D. Revision open reduction and internal fixation of the ulnar Fracture.
Answer: B
In summary, repairing the annular ligament is the recommended treatment for this
patient’s radial head subluxation. Close follow-up and rehabilitation are essential for
optimal recovery.
https://orthoinfo.aaos.org/en/diseases--conditions/radial-head-fractures-of-the-elbow
26. A patient with localized osteosarcoma of the distal femur, returns to see you 1 year after
completing therapy consisting of amputation and chemotherapy. The patient feels well.
Which study will have the greatest yield of detecting recurrence in this patient?
A. Bone scan.
B. Skeletal survey
C. Chest CT
D. MRI of the amputation site
Answer: D
References:
https://www.aafp.org/pubs/afp/issues/2002/0315/p1123.html
https://link.springer.com/chapter/10.1007/978-3-030-43032-0_8
27. A 17-year old with localized osteosarcoma of the distal femur on high-dose methotrexate,
doxorubicin, and cisplatin (MAP) chemotherapy undergoes a limb sparing surgical
resection following 10 weeks of therapy. The pathologist tells you that the percent
necrosis is 80%. You estimate his 5-year survival to fall into which range?
A. 0-20%.
B. 20-40%
C. 40 - 60%
D. 60-80%
E. 80-100%.
Answer: D
Based on the information provided, let’s estimate the 5-year survival rate for this patient
with localized osteosarcoma. Keep in mind that survival rates are estimates and can vary
based on individual factors. Here are the relevant statistics:
● Localized Osteosarcoma:
○ The patient has localized osteosarcoma (no evidence of distant spread).
○ The 5-year relative survival rate for localized osteosarcoma is approximately
76%1.
Given the 80% necrosis observed in this case, we can infer that the patient’s prognosis is
favorable. Therefore, the estimated 5-year survival falls within the range of 60-80%.
Reference: https://ascopubs.org/doi/pdf/10.1200/JGO.17.00241
28. A patient presents with a buttock mass that is completely resected. Staging studies
demonstrate no evidence of disease elsewhere. Biopsy shows a non
rhabdomyosarcomatous soft tissue sarcoma. The following prognostic factors are most
useful in determining whether chemotherapy is indicated;
A. Tumor Size and Tumor Grade
B. Patient age and LDHI
C. Primary site and gender
D. LDH and tumor grade
E. Race and tumor size
Answer: A
● A study by Spunt et al. developed a risk stratification system for NRSTS based on
known prognostic factors1.
● The risk groups were defined as follows:
○ Low-risk: Non-metastatic R0 or R1 low-grade tumor, or ≤5 cm R1 high-grade
tumor.
○ Intermediate-risk: Non-metastatic R0 or R1 >5 cm high-grade tumor, or
unresected tumor of any size or grade.
○ High-risk: Metastatic tumor.
Based on this risk group → metastases, tumor size, and tumor grade → are the momst
useful prognostic factors in determining whether chemotherapy is indicated
References:
https://www.thelancet.com/pdfs/journals/lanonc/PIIS1470-2045%2819%2930672-2.pdf
https://www.tandfonline.com/doi/pdf/10.1080/0284186X.2017.1315173
29. You have a patient with a localized Ewing sarcoma and are considering chemotherapy
options. You know that the addition of ifosfamide and etoposide to a backbone of
vincristine, doxorubicin, and cyclophosphamide has been shown to improve outcome to
the greatest extent for which of the following patients?
A. A 15-year-old with a localized tumor of the tibia
B. A 9-year-old with a localized tumor of the pelvis
C. A 15-year-old with a tumor that is metastatic to bone
D. A 9-year-old with a tumor that is metastatic to lung
Answer: A
Iit was not effective in metastases ewing sarcoma → C and D are excluded.
References:
*Addition of Ifosfamide and Etoposide to Standard Chemotherapy for Ewing's Sarcoma and
Primitive Neuroectodermal Tumor of Bone (nejm.org)
Feasibility of Treating Adults with Ewing or Ewing‐Like Sarcoma with Interval ‐Compressed
Vincristine, Doxorubicin, and Cyclophosphamide Alternating with Ifosfamide and Etoposide
- Lu - 2020 - The Oncologist - Wiley Online Library
Answer: A
References:
https://www.nature.com/articles/s41379-021-00777-y.pdf
https://journals.plos.org/plosone/article?id=10.1371/journal.pone.0210706
31. A patient has pain 2 years after undergoing a metal-on-metal (MOM) left total hip
arthroplasty (THA). Which tests) best correlate with prognosis in the setting of reaction to
metal debris?
A. Erythrocyte sedimentation rate, c-reactive protein, and white blood cell count
B. Serum cobalt and chromium ion levels.
C. MRI with Metal Artifact Reduction Sequence (MARS)
D. CT of pelvis
Answer: B
In the context of a patient experiencing pain 2 years after a metal-on-metal (MOM) left
total hip arthroplasty (THA), several tests can provide valuable information regarding the
prognosis and potential adverse reactions to metal debris. Let’s explore these diagnostic
modalities:
While specialized tests such as metal ion analysis are useful modalities for assessing
MOM hip arthroplasty, over-reliance on any single investigative tool in the clinical
decision-making process should be avoided3. A comprehensive evaluation, including
clinical assessment, imaging, and laboratory tests, is essential for accurate prognosis
and management.
32. Figures 1 and 2 are the recent radiographs of an 82-year-old man with rheumatoid
arthritis who underwent total knee arthroplasty (TKA) 18 years ago.
These radiographs reveal osteolysis with loosening of the tibial component.Aspiration
and laboratory study findings for infection are negative. During the revision TKA,
treatment of tibial bone loss should consist of
Answer: D
In cases of tibial bone loss during revision total knee arthroplasty (rTKA), the treatment
approach depends on the extent of the defect. Let’s explore the options:
Recommendation:
● For tibial bone loss during revision TKA, the choice of treatment depends on the
defect size.
● Small defects may be managed with cement or augments, while larger defects may
require allografts.
● Close collaboration with the surgical team and individualized planning are essential
for successful outcomes.
Remember that the decision should be based on the patient’s specific condition, bone
quality, and available implant options Reference:
https://kneesurgrelatres.biomedcentral.com/articles/10.1186/s43019-022-00158-y
33. A 72-year-old man has had right knee pain for 4 years that is worsening. Three years ago,
he was walker-dependent and received knee injections without any relief (Figure 1) and
then went on to have knee replacement (Figure 2). His symptoms were unchanged and he
underwent revision surgery by another physician (Figure 3). He currently uses a
wheelchair outside the home because of this pain, which is present with ambulation but
not at rest. What is the best next step?
A. evaluation of his ipsilateral hip
B. Revision to a Hinged Prosthesis
C. referral to physical therapy
D. Referral for genicular nerve blockade
Answer: D
https://newsnetwork.mayoclinic.org/discussion/mayo-clinic-q-and-a-pain-after-knee-
replacement-surgery/
34. During revision total knee arthroplasty (TKA), there is significant laxity in 90° of flexion
and 10° short of full extension. Correcting the gap imbalance is best achieved by
A. resecting more tibia.
B. resecting more distal femur to raise joint line, along with resecting more tibia.
C. increasing femoral component size.
D. resecting distal femur and increasing femoral component size
Answer: D
The best approach to correct the gap imbalance during revision total knee arthroplasty
(TKA) depends on the specific situation. Let’s evaluate the options:
35. Figures 1 through 3 are radiographs taken in the emergency department of a 65-year-old
active woman who had a ground level fall and has right hip pain and is unable to bear
weight. To optimize functional outcome and minimize complications, what is the most
appropriate treatment?
Answer: A
36. Figures 1 and 2 are the clinical photographs of a 22-years-old man who was injured 1
years ago when he grasped an opponent during a football game. He experienced
immediate pain and has been unable to close his ring finger into a first since then. He also
has noticed swelling and a painful lump in his palm with attempted forceful gripping.
What is the most likely diagnosis?
Answer: B
The most likely diagnosis for this 22-year-old man with immediate pain, inability to close
his ring finger into a fist, and a painful lump in his palm after grasping an opponent
during a football game is distal rupture of the profundus tendon with entrapment at
the superficialis chiasm .
1. Clinical Presentation:
○ The patient experienced immediate pain after the injury.
○ He is unable to fully extend the finger (cannot close it into a fist).
○ There is swelling and a painful lump in the palm with forceful gripping.
2. Differential Diagnosis:
○ Mallet Finger: Mallet finger typically presents with inability to actively extend
the distal interphalangeal (DIP) joint due to extensor tendon disruption. However,
mallet finger does not involve a painful lump in the palm.
○ Lumbrical Plus Finger: Lumbrical plus finger results from lumbrical muscle
contracture, leading to flexion of the metacarpophalangeal (MCP) joint and
extension of the interphalangeal joints. It does not cause a painful lump in the
palm.
○ Quadrigia: Quadrigia refers to a situation where one flexor tendon is ruptured,
leading to increased load on the adjacent intact tendon. However, it does not
explain the painful lump in the palm.
○ Distal Rupture of the Profundus Tendon with Entrapment at the
Superficialis Chiasm: This condition occurs when the flexor digitorum profundus
(FDP) tendon ruptures distally and becomes entrapped by the intact flexor
digitorum superficialis (FDS) tendon. The lump in the palm corresponds to the
entrapped FDP stump1.
3. Imaging and Confirmation:
○ Ultrasound or MRI can confirm the diagnosis by visualizing the disrupted FDP
tendon and the entrapped stump.
Reference: ajronline.org/doi/pdf/10.2214/ajr.188.3_supplement.0s13
37. What is the most common site of nerve compression in radial tunnel syndrome?
A. Fibrous bands anterior to the radiocapitellar joint
B. Recurrent radial vessels.
C. Medial edge of the extensor carpi radialis brevis (ECRB).
D. Proximal aponeurotic edge of the supinator (arcade of Frohse)
E. Pronator Teres
Answer: D
Reference: Shamrock AG, Das JM. Radial Tunnel Syndrome. [Updated 2023 Aug 14]. In:
StatPearls [Internet]. Treasure Island (FL): StatPearls Publishing; 2024 Jan-. Available
from: https://www.ncbi.nlm.nih.gov/books/NBK555937/
38. A 25-years-ago man has an isolated flexor digitorum profundus laceration just proximal
to the distal interphalangeal (DIP) flexion crease of his ring finger. The tendon ends are
trimmed, removing 10 mm from each and (secondary to fraying) and the tendon repaired.
Four months later, he reports limited finger motion of the long, ring, and small fingers.
Her cannot fully entend his wrist and all joints of the 3 fingers simultaneously. He has full
passive flexion but cannot actively completely close his fingers into a fist. What is the
most likely cause?
A. Quadrigia
B. Intrinsic tightness.
C. Lumbrical plus deformity.
D. Disruption of the tendon repair.
E. Volar plate tightness.
Answer: D
Based on the patient’s history and symptoms, the most likely cause of limited finger
motion and inability to actively close the fingers into a fist after flexor digitorum
profundus (FDP) tendon repair is adhesion formation1. Let’s break down the reasoning:
39. In addition to the anterior oblique ligament, which other structure is an important
stabilizer of the thumb carpometacarpal (CMC) joint when the thumb is in a position of
opposition?
A. Osseous architecture
B. Abductor pollicis brevis
C. Radiovolar joint capsule
D. Dorso Radial Ligament
E. Ulnovolar joint capsule
Answer: D
:https://www.jospt.org/doi/pdf/10.2519/jospt.2003.33.7.386#:~:text=The%20anterior
%20oblique%2C%20radial%20collateral,and%20opposition%20of%20the%20thumb.
40. Dorsal intercalated segment instability (DISI) describes which carpal deformity?
A. Scaphoid extension.
B. Lunate extension
C. Lunate flexion.
D. Triquetral flexion.
E. Scaphoid flexion.
Answer: C
Reference: Smith DK, Gilula LA, Amadio PC. Dorsal lunate tilt (DISI configuration):
sign of scaphoid fracture displacement. Radiology. 1990 Aug;176(2):497-9. doi:
10.1148/radiology.176.2.2367667. PMID: 2367667.
41. Figure above is the MR image of a 36-year-old athlete who is tackled from behind and
falls forward onto his left knee. He has pain, swelling, and stiffness.
Examination includes a moderate effusion, positive quadriceps active test, and normal
lachman test finding.
A. Antero Lateral bundle that is tight in Flexion and posteromedial bundle that is tight in
extension
B. Anterolateral bundle that is tight in extension and a posteromedial bundle that is tight
in flexion
C. Anteromedial bundle that is tight in flexion and a posterolateral bundle that is tight in
extension
D. Anteromedial bundle that is tight in extension and a posterolateral bundle that is tight
in flexion
Answer: D
Given the patient’s symptoms and examination findings, the most likely scenario is that
the anteromedial bundle (AMB) is torn. The AMB is responsible for resisting anterior
tibial translation during extension. When it is injured, the patient may experience
instability and positive quadriceps active test (which stresses the ACL).
This pattern of injury is common in sports-related trauma and can lead to functional
limitations. Further evaluation and management are necessary to address the ACL injury.
42. Figure above is the MR image of a 36-year-old athlete who is tackled from behind and
falls forward onto his left knee. He has pain, swelling, and stiffness.
Examination includes a moderate effusion, positive quadriceps active test, and normal
lachman test finding.
Left untreated, injury to this structure most likely will lead to degenerative changes In
Answer: A
The injury described in the scenario most likely involves the anterior cruciate ligament
(ACL). Left untreated, ACL injury can lead to degenerative changes in the medial and lateral
compartments of the knee joint. These compartments are particularly affected due to altered
biomechanics and instability resulting from ACL deficiency. It’s essential to address ACL
injuries promptly to prevent long-term complications.
Reference: https://www.spine.md/insights/stages-of-degenerative-disc-disease
43. Figure above is the MR image of a 36-year-old athlete who is tackled from behind and
falls forward onto his left knee. He has pain, swelling, and stiffness.
Examination includes a moderate effusion, positive quadriceps active test, and normal
lachman test finding.
If the patient chooses surgical reconstruction, he should be advised that, when compared
to a transtibial technique, the tibial inlay technique has been shown to provide
A. Stronger initial graft fixation
B. More anatomic positioning of tibial fixation
C. More natural knee kinematics during deep flexion
D. More graft protection during Cyclic Loading
Answer: B
When comparing the transtibial technique to the tibial inlay technique for anterior
cruciate ligament (ACL) reconstruction, the tibial inlay technique has been shown to
provide more anatomic positioning of tibial fixation. Let me explain further:
1. Transtibial Technique:
○ In the transtibial technique, the femoral tunnel is created through the tibial tunnel.
○ However, this method is criticized for its inability to consistently restore the
anatomic femoral tunnel placement1.
○ It may not achieve optimal positioning of the graft.
2. Tibial Inlay Technique:
○ The tibial inlay technique involves creating a tibial tunnel and placing the graft in
a more anatomically accurate position.
○ By directly addressing the tibial attachment site, it aims to restore natural knee
kinematics and improve stability2.
○ This technique provides better graft protection during cyclic loading.
Therefore, if the patient chooses surgical reconstruction, the tibial inlay technique may offer
advantages in terms of anatomical positioning and graft protection
Reference: https://www.drbethshubinstein.com/wp-content/uploads/2016/11/Comparison-of-
Tibial-Inlay-Versus-Transtibial-Techniques-for-Isolated-Posterior-Cruciate-Ligament-
Reconstruction-Minimum-2-Year-Follow-up.pdf
44. Figure above is the MR image of a 36-year-old athlete who is tackled from behind and
falls forward onto his left knee. He has pain, swelling, and stiffness.
Examination includes a moderate effusion, positive quadriceps active test, and normal
lachman test finding.
This patient elects nonsurgical treatment and later experiences persistent instability.
Examination reveals an asymmetric. Dial test finding and a varus thrust during abulation.
Which osteotomy and correction appropiately addresses this chronic instability pattern?
Answer: D
Given the chronic instability pattern described in the patient, the appropriate osteotomy
and correction would be a high tibial osteotomy (HTO) with a closing lateral wedge and
decreased tibial slope. This procedure aims to address the varus thrust during ambulation
and restore proper alignment of the knee joint.
https://link.springer.com/referenceworkentry/10.1007/978-3-642-36801-1_130-3
45. Figure above is the MR image of a 36-year-old athlete who is tackled from behind and
falls forward onto his left knee. He has pain, swelling, and stiffness.
Examination includes a moderate effusion, positive quadriceps active test, and normal
lachman test finding.
Risk for vascular injury during transtibial drilling for reconstruction of this injury is
increased by
A. Accessory incisions
B. Use of tapered drill bits
C. Use of oscillating drills
D. Greater Knee Extension
Answer: B
When performing transtibial drilling for reconstruction of the anterior cruciate ligament
(ACL), the risk for vascular injury is increased by using tapered drill bits. Tapered drill
bits can inadvertently penetrate the posterior cortex of the tibia, potentially damaging
nearby vascular structures. It’s essential to exercise caution during the drilling process to
minimize this risk.
Reference: https://link.springer.com/article/10.1007/s00264-017-3554-7
46. Figure above is the MR image of a 14-year-old football player who injured his right knee
during a game. He describes feeling a "pop" and then he needed help walking off the
field. His knee is visibly swollen.
Knee range of motion is between O degrees and 70 degrees. What is the most appropriate
treatment option?
A. Open reduction and internal fixation of the lateral condyle
B. Microfracture of the chondral defect
C. Immediate anterior cruciate ligament (ACL) reconstruction
D. Delayed ACL Reconstruction
Answer: C
Given the patient’s history of acute knee injury, visible swelling, and limited range of
motion (between 0 degrees and 70 degrees), the most appropriate treatment option is
immediate anterior cruciate ligament (ACL) reconstruction. ACL reconstruction is
indicated to restore knee stability and prevent further damage to the joint.
Reference: https://link.springer.com/article/10.1007/s12178-023-09875-7
47. Figure above is the MR image of a 14-year-old football player who injured his right knee
during a game. He describes feeling a "pop" and then he needed help walking off the
field. His knee is visibly swollen.
The patient has no postsurgical complications and begins physical therapy rehabilitation.
The boy and his parents stress they want to get the therapy over with as fast as possible to
expedite his return to sports, and the surgeon and rehabilitation team consider their
request. Compared to non accelerated rehabilitation, patients who follow an early
accelerated rehabilitation protocol
A. Increased laxity
B. Increased risk for graft failure
C. No differences in Long Term Results
D. Lower knee injury and osteoarthritis outcome scores (KOOS)
Answer: C
When comparing accelerated rehabilitation to conventional non-accelerated protocols after
anterior cruciate ligament (ACL) reconstruction, it’s essential to consider the potential
effects. Let’s explore the outcomes associated with early accelerated rehabilitation:
1. Increased Laxity:
○ Accelerated rehabilitation may lead to increased joint laxity (instability).
○ This could affect knee stability during activities and potentially increase the risk of
reinjury.
2. Increased Risk for Graft Failure:
○ While accelerated protocols aim for faster recovery, there is a concern that early
return to high-level activities may stress the graft.
○ Premature loading and aggressive rehabilitation could potentially increase the risk
of graft failure.
3. No Differences in Long-Term Results:
○ Some studies suggest that long-term outcomes (e.g., functional scores, return to
sports) are similar between accelerated and non-accelerated protocols1.
○ However, this remains an area of ongoing research.
4. Lower Knee Injury and Osteoarthritis Outcome Scores (KOOS):
○ Accelerated rehabilitation may lead to improved short-term functional scores (such
as KOOS) due to faster progress.
○ However, the long-term impact on knee function and osteoarthritis risk is not fully
understood.
Recommendation:
● While accelerated rehabilitation may expedite early recovery, it’s crucial to balance the
benefits with potential risks.
● Individualized patient factors, graft type, and surgeon expertise should guide the decision.
● Close monitoring and gradual progression are essential to optimize outcomes.
Remember that the choice between accelerated and non-accelerated rehabilitation should be
based on the patient’s specific needs and goals
Reference: https://bjsm.bmj.com/content/57/9/500.long
48. A 32-year-old volleyball player has dull posterior shoulder pain. An examination reveals
moderate external rotation weakness with his arm at his side, but normal strength on
supraspinatus isolation. Deltoid and supraspinatus bulk appear normal, although there
appears to be mild infraspinatus atrophy. Sensation is normal throughout the shoulder and
shoulder girdle. What is the most likely diagnosis?
A. Calcified transverse scapular ligament.
B. Parsonage-Turner syndrome.
C. Spinoglenoid notch cyst
D. Quadrilateral space syndrome
Answer: B
Based on the information provided, the most likely diagnosis for this 32-year-old
volleyball player with dull posterior shoulder pain and the described examination findings
is Parsonage-Turner syndrome1. Parsonage-Turner syndrome, also known as neuralgic
amyotrophy, is characterized by sudden-onset shoulder pain followed by weakness and
atrophy of the shoulder girdle muscles. It typically affects the suprascapular nerve,
causing weakness in the infraspinatus and supraspinatus muscles. The deltoid muscle is
usually spared, which aligns with the normal strength observed during supraspinatus
isolation in this case. Sensation remains intact, ruling out other potential causes such as
nerve compression or impingement. The other options (calcified transverse scapular
ligament, spinoglenoid notch cyst, and quadrilateral space syndrome) do not fit the
clinical presentation described here
Reference: https://my.clevelandclinic.org/health/diseases/15390-parsonage-turner-
syndrome
49. A 16-year-old swimmer has right shoulder pain with activity. She describes the continued
sensation that her shoulder is "loose". She has been in physical therapy for 7 months to
work on strengthening the muscles around her shoulder and scapula. She denies being
able to voluntarily dislocate her shoulder. Upon examination, you can feel the humeral
head slide over the glenoid rim both anteriorly and posteriorly with the load and shift test.
She has a grade Ill sulcus sign. What is the most appropriate next step?
A. Arthroscopic superior labrum anterior to superior repair.
B. Arthroscopic bankart repair.
C. Latarjet procedure.
D. Capsulorrhaphy
Answer: B
Given the clinical presentation of a 16-year-old swimmer with right shoulder pain, a
sensation of shoulder “looseness,” and a history of physical therapy to strengthen the
shoulder muscles, we need to consider the most likely diagnosis and appropriate
management.
● Humeral head sliding over the glenoid rim both anteriorly and posteriorly during
the load and shift test indicates shoulder instability.
● The grade III sulcus sign suggests glenohumeral joint laxity.
Considering these findings, the most appropriate next step would be to address the
shoulder instability. Let’s evaluate the options:
Therefore, the most appropriate next step for this patient would be b. Arthroscopic
Bankart repair to address the anterior shoulder instability and prevent further dislocations.
This procedure aims to restore stability by repairing the torn labrum and tightening the
capsule.
Reference: Swimmer's Shoulder: Symptoms, Tests, Treatment & Recovery
(clevelandclinic.org)
50. Figures 28a and 28b are the MR of a 30-year-old man who has right shoulder pain and
difficulty throwing a football. His history includes a shoulder injury from a skiing
accident 2 years ago. He has not had a recent shoulder injury. Which shoulder motion is
most likely to demonstrate weakness?
A. Shoulder abduction
B. Shoulder internal rotation
C. Shoulder External rotation
D. Shoulder adduction
Answer: C
Given the patient’s history of a skiing accident resulting in shoulder injury, let’s consider
the most likely motion to demonstrate weakness:
● Shoulder External Rotation
Explanation:
● The skiing accident may have caused trauma to the shoulder joint or rotator cuff.
● External rotation of the shoulder involves the infraspinatus and teres minor muscles,
which are part of the rotator cuff.
● Weakness in external rotation can be indicative of rotator cuff pathology or injury1.
Therefore, the most likely motion to demonstrate weakness in this case is shoulder
external rotation
Reference: https://orthoglobe.org/skiing-injuries-of-the-upper-limb/
51. Figures 1 and 2 are the clinical photographs of a 2-month-old infant with a foot
deformity. The parents have been doing stretching exercises at home with some
improvement in foot position. What is the most appropriate course of treatment?
A. Institute serial casting and schedule for surgical intervention
B. Provide parental reassurance and follow-up as needed
C. Commence brace treatment until deformity is corrected
D. Obtain AP and lateral tibia views, and plan follow-up until maturity.
Answer: A
Reference: https://www.orthobullets.com/pediatrics/4067/calcaneovalgus-foot
52. A 5-year-old boy develops immediate left elbow pain and swelling following a fall from
his hover board. His fracture is demonstrated in Figures 1 and 2. He is taken to the
operating room and treated with open reduction and unburied pin fixation. The main
advantage of unburied pin fixation compared with buried pin fixation method is?
A. unburied pins require more return trips to the operating room.
B. union rate is higher with unburied pins.
C. infection rate is lower with unburied pins.
D. un-buried pins are associated with lower patient Costs.
Answer: C
53. Figures 1 and 2 are the radiographs of a 5-year-old boy who was treated for a
nondisplaced ulna fracture. Eight months later, he complains of a painful prominence
over the elbow, causing pain with direct trauma during activity, occurring for several
months. What is the best next step in the management of this patient?
A. Excision of the radial head
B. Osteotomy of the Ulnar Deformity
C. Annular ligament reconstruction
D. Osteotomy of the ulna and radius
Answer: A
Given the patient’s history of a nondisplaced ulna fracture and the subsequent
development of a painful prominence over the elbow, the best next step in management
would be excision of the radial head. Let me explain why:
54. A 15-year-old football player sustains a concussion on the field. His mother and coach are
asking when it will be safe for him to begin a return-to-play program. For the best
outcome, he should be advised to wait until he
A. is confident he has fully recovered.
B. completes a week of complete brain rest.
C. undergoes a brain MRI showing no evidence of injury.
D. is completely symptom-free-at rest.
Answer: D
Reference: https://pubmed.ncbi.nlm.nih.gov/22688215/#:~:text=Practice%20guidelines
%20universally%20recommend%20an,daily%20life%20or%20military%20service
55. Figures 1 and 2 are the radiographs of a 6-year-old boy who came to the emergency
department with a twisting injury to the right leg sustained while skiing.
He is non weight bearing, has no pain with passive motion of his ankle and toes, and
remains neurovascularly intact. What is the most appropriate next step in the management
of his injury?
A. Flexible Intramedullary nailing
B. Long Leg Casting
C. External fixation
D. Plate and Screw fixation
Answer: B
The most appropriate next step in the management of this 6-year-old boy with a total
fracture cruris (fracture involving the lower leg) is Long Leg Casting. Let’s discuss the
rationale:
1. Clinical Presentation:
○ The patient is non-weight-bearing.
○ There is no pain with passive motion of the ankle and toes.
○ Neurovascular status remains intact.
2. Treatment Options:
○ Long Leg Casting: This conservative approach involves immobilizing the leg
with a cast to allow healing and prevent further injury. It is suitable for stable
fractures or soft tissue injuries.
○ External Fixation: External fixation devices are typically used for more severe
fractures or unstable injuries.
○ Flexible Intramedullary Nailing and Plate and Screw Fixation are surgical
options and are not indicated in this case of a stable injury.
● Considering the patient’s age, presentation, and the absence of significant symptoms
or neurovascular compromise, Long Leg Casting is the most appropriate next step.
Reference: The Principles of Pediatric Fracture and Trauma Care | Musculoskeletal Key
56. Figures 1 and 2 are the clinical photograph and radiograph of a newborn who is seen for
evaluation of his feet. A rigid rocker bottom foot is present with dorsal foot crease. The
most appropriate next step in management is
A. Serial casting with talonavicular reduction and fixation and Achilles tenotomy.
B. operative release at age 18 months.
C. tendon transfer after age 18 months.
D. observation, physical therapy, and transition into an ankle foot orthosis.
Answer: A
Answer: B
The most likely angular deformity associated with transphyseal anterior cruciate ligament
(ACL) reconstruction using a hamstring autograft in skeletally immature patients is
valgus deformity12. This occurs due to tensioning of the graft, which can lead to
significant valgus (outward angulation) of the distal femur and varus (inward angulation)
of the proximal tibia. It’s essential to consider this risk when choosing the graft type and
managing pediatric ACL injuries.
Reference: 2325967119872450 (sagepub.com)
58. A 4-year-old girl comes to the emergency department with a 3-day history of fever, a
limp, and left knee pain and swelling. There is no history of recent trauma.
Her temperature is 102.6° F. Her left knee is warm, erythematous, and tender with
restricted range of motion. Her WBC count is 14,500, ESR is 72, CRP level is 10.2.
What is the most appropriate next step for management of this patient?
A. Broad-spectrum antibiotics
B. MRI scan of left knee.
C. Aspiration of left knee.
D. Emergent incision and drainage
Answer: A
Diagnosis: Septic arthritis
Management:
○ Septic arthritis is an orthopedic emergency.
○ Treatment involves prompt antibiotic therapy to eradicate the infection.
○ Surgical drainage and joint lavage may be necessary.
○ Early diagnosis is crucial to prevent long-term joint damage.
Reference: Septic Arthritis (Infectious Arthritis) in Children - Stanford Medicine
Children's Health (stanfordchildrens.org)
59. A limping 13-year-old boy is seen in the emergency department several months after
returning from summer camp complaining of 5 days of progressive knee swelling and
low-grade fever. The patient does not recall a rash or tick bite but notes that several
friends had ticks removed while at camp. A physical examination reveals a large right
knee effusion, moderately noxious arc of passive motion from 30-95°, and a fever of
38.1°C. Laboratory evaluation evidences a WBC count of 12.3, ESR of 42, and CRP level
of 4.4. Knee aspirate obtains 40 cc of cloudy synovial fluid with cell count of 64,000
WBC with 72% PMNs. What is the most appropriate course of treatment?
A. Emergent arthroscopic lavage
B. Observation and IV non-steroidal anti-inflammatory drugs (NSAIDs) with repeat
aspiration in the morning
C. Emergent MRI with contrast of the right knee
D. Serologic Lyme Titer Testing and Initiation of Doxycycline and NSAIDs
Answer: D
1. Clinical Presentation:
○ The patient has a history of being in a tick-infested area (summer camp).
○ Knee swelling, fever, and a moderately noxious arc of passive motion are
suggestive of an inflammatory process.
2. Diagnostic Considerations:
○ The cloudy synovial fluid with elevated white blood cell count (64,000 WBC
with 72% PMNs) indicates an inflammatory joint effusion.
○ Given the exposure history, Lyme arthritis should be considered.
3. Lyme Arthritis:
○ Lyme arthritis is caused by Borrelia burgdorferi, transmitted by ticks (Ixodes
scapularis).
○ It typically presents with monoarticular or oligoarticular joint involvement,
especially the knee.
○ Symptoms include joint swelling, pain, and fever.
○ Diagnosis is confirmed by serologic testing (Lyme titer).
4. Treatment:
○ Doxycycline is the preferred antibiotic for treating Lyme arthritis in children.
○ NSAIDs can help manage pain and inflammation.
60. Figures 1 and 2 are the radiographs of a 10-year-old girl who injured her elbow in an all-
terrain vehicle accident. Figures 3 and 4 are radiographs of the elbow following closed
manipulation. What is the best next step in management?
A. Immobilization in a long arm cast
B. Percutaneous reduction and pin fixation, if required.
C. Open reduction and pin fixation, if required
D. Open reduction and screw-plate fixation
Answer: B
61. Figures 1 through 5 are the clinical photographs and radiographs of a 7-year-old boy who
has a history of a left elbow supracondylar fracture treated with closed reduction and
pinning one year ago. He is referred due to a left arm deformity, the family is notching
over the last year. He has no pain. What most accurately describes the nature of this
deformity?
A. A cosmetic problem
B. It could lead to tardy posteromedial rotatory instabillity
C. A varus, extension, and Rotational Deformity of the distal humerus
D. It has no long term sequelae
E. Hyperlaxity of the join
Answer: C
The nature of the deformity seen in the clinical photographs and radiographs of the 7-year-
old boy with a history of a left elbow supracondylar fracture is a varus, extension, and
rotational deformity of the distal humerus.
62. Figures 1 and 2 are the radiographs of an 8-year-old boy who was brought to the
emergency department after failing from monkey bars. The clinical finding in figure 3 is
caused by impingement of the proximal bone fragment on which structure?
A. Brachialis Fascia
B. Biceps tendon
C. Median nerve
D. Lacertus fibrosus
Answer: A
The clinical finding seen in Figure 3 is caused by impingement of the proximal bone
fragment on the brachialis fascia. This impingement can lead to symptoms and functional
limitations.
Reference: https://www.ajronline.org/doi/pdf/10.2214/AJR.06.0921
63. A 5-year-old boy develops immediate left elbow pain and swelling following a fall from
his hover board. His fracture is demonstrated in Figures 1 and 2. He is taken to the
operating room and treated with open reduction and unburied pin fixation. The main
advantage of unburied pin fixation compared with buried pin fixation method is
Answer: C
The main advantage of unburied pin fixation compared to the buried pin fixation method
is that unburied pins are associated with a lower infection rate. When pins are left
exposed (unburied), it allows for better monitoring, early detection of any signs of
infection, and prompt management if needed. In contrast, buried pins are covered by soft
tissue or bone, making it harder to assess their condition and increasing the risk of
infection going unnoticed.
64. The four anatomic sites with hightest risk of concominant pediatric septic arthritis and
osteomyelitis are
A. Proximal humerus, distal femur, proximal femur, and distal forearm.
B. Proximal humerus, proximal ulna, proximal femur, and proximal tibia.
C. Proximal humerus, proximal radius, proximal femur, and distal fibula
D. Proximal radius, distal forearm, proximal femur, and distal tibia.
E. Proximal humerus, proximal radius, proximal femur, and distal tibia.
Answer: D
References:
https://link.springer.com/referenceworkentry/10.1007/978-3-030-38095-3_87-1
https://journals.lww.com/pedorthopaedics/Abstract/2013/06000/
Concurrent_Septic_Arthritis_and_Osteomyelitis_in.21.aspx
https://journals.lww.com/pedorthopaedics/Abstract/2017/12000/
Osteomyelitis_is_Commonly_Associated_With_Septic.7.aspx
https://link.springer.com/article/10.1007/s11908-019-0678-5
https://publications.aap.org/pediatricsinreview/article/41/3/120/35383/Acute-
Hematogenous-Bacterial-Osteoarticular
65. A 7-year-old girl comes to the emergency department after falling from monkey bars and
landing on her outstretched hand. Radiographs shows an extension Gartland type III
supracondylar fracture of the humerus. Immediately following the fall, she complains of
pain and swelling of the right elbow. Examination reveals absent radial and ulnar pulses
and associated anterior interosseous nerve (AIN) injury. The hand is pink with brisk
capillary refill. Pulses are undetectable by Doppler ultrasonography. What is the best next
step in the management of this patient?
A. Immediate closed reduction in the emergency department under sedation
B. Emergent closed reduction and Percutaneous Pinning
C. Open reduction (OR) and exploration of the brachial artery
D. CT angiography for evaluation of the brachial artery
E. Closed observation in the emergency ward
Answer: B
Given the clinical scenario of a 7-year-old girl with an extension Gartland type III
supracondylar fracture of the humerus, associated anterior interosseous nerve (AIN)
injury, and absent radial and ulnar pulses, let’s evaluate the best next step in management.
Therefore, the best next step in the management of this patient is b. Emergent closed
reduction and percutaneous pinning (CRPP). The goal is to stabilize the fracture and
restore blood flow to the affected limb. Close monitoring of pulses and neurovascular
status is essential during and after the procedure
Reference: Supracondylar Fracture - Pediatric - Pediatrics - Orthobullets
Answer: B
The most likely angular deformity associated with transphyseal anterior cruciate ligament
(ACL) reconstruction using a hamstring autograft in skeletally immature patients is
valgus deformity12. This occurs due to tensioning of the graft, which can lead to
significant valgus (outward angulation) of the distal femur and varus (inward angulation)
of the proximal tibia. It’s essential to consider this risk when choosing the graft type and
managing pediatric ACL injuries.
Reference: 2325967119872450 (sagepub.com)
67. What is the most common complication after successful treatment of distal femoral
physeal fracture?
A. Nonunion
B. Compartment syndrome
C. Knee stiffness
D. Premature Physeal Closure
E. Physeal overgrowth
Answer: D
The most common complication after successful treatment of a distal femoral physeal
fracture is premature physeal closure. This occurs when the growth plate (physis) closes
prematurely, potentially leading to limb length discrepancies and angular deformities.
Although surgical management aims to stabilize the fracture and promote healing, the risk
of growth disturbance remains.
Reference: https://www.ncbi.nlm.nih.gov/pmc/articles/PMC10348013/
68. A 4-year-old girl comes to the emergency department with a 3-day history of fever, a
limp and left knee pain and swelling. There is no history of recent trauma.
Her temperature is 102.6Her left knee is warm, erythematous and tender with restricted
range of motion. Her WBC count is 14.500, ESR is 72, CRP level is 10.2.
What is the most appropriate next step for management of this patient?
A. Broad-spectrum antibiotics.
B. MRI scan of left knee.
C. Aspiration of left knee.
D. Emergent incision and drainage
E. Sonography test for knee joint
Answer: D
Given the 4-year-old girl’s symptoms of fever, limp, left knee pain, swelling, and
restricted range of motion, the most appropriate next step for management is aspiration of
the left knee. Let me explain why:
1. Septic Arthritis (Acute Bacterial Arthritis):
○ The combination of fever, warm, erythematous, and tender knee, along with
restricted range of motion, raises concern for septic arthritis.
○ Septic arthritis is an urgent condition that requires prompt evaluation and
treatment.
2. Aspiration of the Left Knee:
○ Aspiration involves removing synovial fluid from the joint for analysis.
○ It helps confirm the diagnosis (presence of pus or elevated white blood cell
count) and guide appropriate treatment.
○ Joint fluid analysis includes cell count, Gram stain, culture, and sensitivity
testing.
3. Other Considerations:
○ Broad-spectrum antibiotics are essential, but the diagnosis should be confirmed
before starting antibiotics.
○ MRI or sonography may be useful later in the evaluation, but aspiration is the
initial step.
Recommendation:
69. Figures 1 through 4 are the injury radiographs and postsurgical open treatment
radiographs of a 13-year-old girl who fell while on a trampoline and sustained an injury
to her right-dominant elbow. The skin Is closed and she has normal vascular and
neurologic examination findings. Which complication most likely could occur as a result
of this injury and treatment?
A. Compartment syndrome.
B. Loss of elbow motion
C. Avascular necrosis (AVN) of the radial head.
D. Nonunion of the fracture site.
Answer: C
Given the injury radiographs and postsurgical open treatment radiographs of the 13-year-
old girl with a right-dominant elbow injury, the most likely complication that could occur
as a result of this injury and treatment is avascular necrosis (AVN) of the radial head.
Avascular necrosis refers to the loss of blood supply to the bone, leading to bone tissue
death. In this case, the radial head may be at risk due to the injury and surgical
intervention. Early recognition and management are crucial to prevent long-term
complications
Reference: https://www.hand.theclinics.com/article/S0749-0712%2815%2900078-5/pdf
70. Which factor leads to the worst long-term prognosis in slipped capital femoral epiphysis
(SCFE), most likely requiring total hip arthroplasty (THA)?
A. Avascular Necrosis (AVN)
B. Age at onset of SCFE.
C. Slip severity or slip angle.
D. Femoral acetabular impingement (FA) with degenerative changes
Answer: A
71. A 44-year-old male presents with a fungating mass of the left distal leg and hindfoot.
Figures 1 and 2 are the axial and coronal MRI scans of the lesion which measures >10 cm
but <15 cm in maximal dimension. Physical examination shows a palpable inguinal
lymph node. CT scans of the chest, abdomen, and pelvis show no pulmonary metastases
but confirm the enlarged lymph node (Figure 3). Biopsy of the node shows synovial
sarcoma. According to the current American Joint Committee on Cancer (AJCC) 8th
Edition staging system, this patient's cancer is at what stage?
A. IB
B. IIIA
C. IIIB
D. IV
Answer: D
72. A coronal MRI scan of an 82-year-old male with left hip pain exacerbated by weight
bearing. Biopsy shows diffuse large B-cell lymphoma. Staging shows no other sites of
disease. Which of the following is the most appropriate treatment?
A. Radiotherapy only
B. Prophylactic fixation and radiotherapy
C. Wide resection and reconstruction
D. Prophylactic Fixation and Chemotherapy
Answer: D
Diffuse Large B-cell Lymphoma (DLBCL) is an aggressive but potentially curable
lymphoma subtype. Given the patient’s age and diagnosis, treatment decisions should
consider both fitness status and comorbidities. Here are the relevant treatment options:
1. Standard Immunochemotherapy:
○ Rituximab-CHOP Regimen: The standard chemotherapy partner for DLBCL is
the CHOP regimen (cyclophosphamide, doxorubicin, vincristine, and
prednisone) combined with the monoclonal antibody rituximab.
○ Fit older patients can benefit from this standard immunochemotherapy, which has
shown good results in terms of cure rates12.
2. New Therapies:
○ Several new therapies have indicated promising results in relapsed/refractory
DLBCL patients, particularly those not eligible for transplantation:
■ Polatuzumab Vedotin: An antibody-drug conjugate targeting CD79b.
■ Tafasitamab: A monoclonal antibody targeting CD19.
■ Bispecific Antibodies: These agents engage both tumor cells and immune
cells to enhance antitumor activity.
○ Some of these new drugs have been tested as single agents or in combinations as
first-line treatment, aiming to improve outcomes compared to traditional
chemotherapy.
○ If preliminary efficacy and safety data are confirmed in future clinical trials, a
chemo-free immunotherapeutic approach could become an alternative option,
even for frail patients3.
In summary, the choice of treatment should be individualized based on the patient’s fitness
status, comorbidities, and disease characteristics. Consultation with an oncologist or
hematologist is essential to determine the most appropriate therapeutic approach for this
elderly patient with DLBCL.
Reference: How I treat elderly patients with diffuse large B-cell lymphoma | Blood |
American Society of Hematology (ashpublications.org)
73. 75-year-old man is transferred in for management of an infected left total knee
arthroplasty (TKA). He has had two irrigation and debridement with polyethylene linier
exchanges for a resistant enterococcus bacterium that has been cultured from
intraoperative specimens during these procedures. He now has an open wound reported
50% deficits of the patella tendon from the prior debridement. The patients can straight
leg raise but is clearly weak. His range of motion is 80 to 1000 . His past medical history
includes chronic lymphocytic leukaemia (CLL) that is in remission and non-insulin
dependent diabetes mellitus (NIDDM) that is well controlled with HgbA1c of -6. What is
the best option for treating this patient's periprosthetic joint infection?
A. Two stage revision TKA with a static spacer and plastic surgery wound coverage
B. One stage revision TKA with cemented components and plastic surgery wound
coverage.
C. Two stage revision TKA with a dynamic spacer and plastic surgery wound coveras
D. One stage revision TKA with cementless components and primary closure.
Answer: B
The management of periprosthetic joint infection (PJI) involves both surgical intervention
and antimicrobial therapy. The approach depends on various factors, including the timing
and microbiology of the infection, the condition of the joint and implant, the quality of
the soft tissue envelope, and individual patient circumstances1.
Given the patient’s history and clinical presentation, let’s evaluate the treatment options:
1. Two-stage revision TKA with a static spacer and plastic surgery wound coverage:
This approach involves removing the infected prosthesis, placing a temporary spacer
(static or dynamic), and allowing the infection to resolve before performing a second-
stage revision with a new prosthesis. However, this option may not be ideal for a
patient with significant soft tissue deficits.
2. One-stage revision TKA with cemented components and plastic surgery wound
coverage: In one-stage revision, the infected prosthesis is removed, and a new
prosthesis is implanted during the same procedure. This approach is suitable for
patients with stable implants and good soft tissue coverage. However, the presence of
significant soft tissue deficits may limit its effectiveness.
3. Two-stage revision TKA with a dynamic spacer and plastic surgery wound coverage:
Similar to the first option, this approach involves a two-stage revision with a dynamic
spacer. The dynamic spacer allows for better joint mobility during the interim period.
However, the decision between static and dynamic spacers depends on the surgeon’s
preference and patient-specific factors.
4. One-stage revision TKA with cementless components and primary closure: In this
approach, the infected prosthesis is removed, and a new prosthesis (cementless) is
implanted during the same surgery. The wound is closed primarily. This option is
suitable for patients with stable implants and adequate soft tissue coverage.
Considering the patient’s weakened state and the 50% deficits in the patellar tendon, a
one-stage revision with cemented components and plastic surgery wound coverage
(Option B) may be the most appropriate choice.
Reference:Periprosthetic Joint Infection: Consensus and Guidelines | SpringerLink
74. 74.Figures 1 and 2 are radiographs of a 72 year -old man 5 weeks after a right total knee
arthroplasty (TKA). The patient has had continued drainage from a large hematoma in his
right knee, despite an irrigation and debridement 4 weeks prior.
His physical examination is notable for swelling and erythema with active purulent
drainage. Prior operative cultures were negative for growth and repeat aspiration was
negative for growth. What is the best next step?
Answer: D
Given the patient’s continued drainage from a large hematoma, swelling, erythema, and
active purulent drainage despite prior irrigation and debridement, the best next step would be
removal of components and a knee fusion with antibiotic beads
75. Pulsatile bleeding is encountered after placing a retractor anterior to the acetabulum while
exposing for reaming during total hip arthroplasty (THA). What vascular structure is
likely affected?
A. Ascending branches of the lateral femoral circumflex artery.
B. Obturator artery.
C. Superior gluteal artery.
D. External iliac artery
Answer: B
The vascular structure likely affected during total hip arthroplasty (THA) when pulsatile
bleeding is encountered after placing a retractor anterior to the acetabulum is the
obturator artery. The obturator artery runs in close proximity to the acetabulum and can
be at risk during surgical exposure. Proper identification and management are crucial to
prevent complications
76. Figures 1 and 2 are the preoperative radiographs of a 75-year old woman with a right hip
osteoarthritis who presents for a right total hip arthroplasty (THA). During the intake
history and physical, the patients discloses that she has been treated with bisphosphonates
by her primary care physician. To reduce the risk of a periprosthetic fracture and optimize
long term survivorship of the THA, what is the best option for acetabular and femoral
implant selection?
A. Cemented acetabular component and a cementless femoral component.
B. Cementless acetabular component and a Cemented femoral component
C. Cementless acetabular component and a cementless femoral component
D. Cemented acetabular component and a cemented femoral component
Answer: A
Acetabular Component Selection:
a. The choice of acetabular component is crucial for the success of a total hip
arthroplasty (THA). There are different types of acetabular components available,
including cemented and cementless options.
b. Cemented acetabular components have been used historically, but newer evidence
suggests that cementless components are suitable for younger patients due to their
potential for better long-term survivorship1.
c. Considering the patient’s age (75 years) and the goal of optimizing long-term
survivorship, a cementless acetabular component may be a better choice.
Cementless components rely on bone ingrowth for stability and can provide durable
results over time.
Femoral Component Selection:
d. The femoral component also plays a significant role in THA. It’s essential to consider
factors such as bone quality, patient age, and functional demand.
e. While there are various options for femoral components, the choice often comes down
to the type of articulation (bearing surface). The most common combination is a
metal femoral head with a polyethylene acetabular cup (metal-on-polyethylene) 2.
f. However, the choice of femoral component material (cemented or cementless)
depends on several factors, including surgeon experience, patient characteristics, and
implant longevity.
g. Given the patient’s age and the desire for long-term success, a cementless femoral
component may be preferable. Cementless stems allow for better bone preservation
and potential for biological fixation.
In summary, the recommended option for this patient would be a cementless acetabular
component combined with a cementless femoral component.
Reference: https://www.orthobullets.com/recon/5013/tha-periprosthetic-fracture
77. An 88-year-old man presents with a persistent left thigh pain after revision total hip
arthroplasty. He initially sustained a fatigue fracture of a cylindrical distallyfixed stem
(Figure 1) that was treated with an extended trochanteric osteotomy, trephining and
revision femoral surgery. This implant subsequently subsided and a loner modular,
tapered stem was inserted (Figure 2). Over the next 6 months, he developed worsening
thigh pain and now presents with the radiographs figures 3 and 4. What is the appropriate
treatment option?
Reference:
https://journals.lww.com/jaaos/Abstract/2002/11000/Thigh_Pain_After_Cementless_Tota
l_Hip.2.aspx
78. Figure 1 is the radiograph of an otherwise healthy 68-year-old man with a 4-year history
if increasing global left knee pain. He has noticed stiffness, and despite physical therapy,
bracing and non-steroidal anti-inflammatory drugs, he has continued to develop
worsening symptoms and progression in his deformity.
Physical examination demonstrates of 80 degree of flexion and a 10 degree flexion
contracture.
What is the best next step?
A. Manipulation under anesthesia
B. Left Total Knee Arthroplasty (ТКА)
C. Stem cell injection
D. Unicompartmental knee arthroplasty in the lateral compartment
Answer: B
Reference: (PDF) Highly purified chondroitin sulfate: a literature review on clinical
efficacy and pharmacoeconomic aspects in osteoarthritis treatment (researchgate.net)
79. A 68-year-old man presents 15 days after left total hip arthroplasty with increasing pain
and subjective fevers for the last 3 days. Physical exam reveals a healing surgical incision
with moderate erythema and no drainage and pain with range of motion of the hip. ESR is
44 and CRP is 32.4. His aspiration respiration reveals 8000 WBX. 80%
polymorphonuclear leukocytes (PMN) and two cultures positive for Cutibacterium acnes.
Based on the recommendation of the 2018 Second International Consensus Meeting on
Musculoskeletal Infection, what is the most appropriate treatment?
A. Debridement and Implant Retention (DAIR).
B. Resection arthroplasty due to chronic infection
C. Repeat aspiration due to equivocal result
D. Non-surgical treatment of C. acnes as a Nonpathological organism
Answer: A
Based on the recommendations from the 2018 Second International Consensus Meeting
on Musculoskeletal Infection, the most appropriate treatment for this patient’s
periprosthetic joint infection (PJI) caused by Cutibacterium acnes (formerly known as
Propionibacterium acnes) is Debridement and Implant Retention (DAIR)12. Let’s break
down the reasoning behind this choice:
80. A 72-year-old woman has a painful right hip and left hip issues are discovered on the
radiographs shown in figures 1 and 2. An arthroplasty was done 24 years previously. Her
left hip is pain-free, but she reports occasional clicking and grinding on the left side. She
wishes to avoid major revision surgery. Considering this, what is the best next step to
address the left hip?
A. Repeat radiographs at age 75.
B. Intraarticular injection with bone marrow aspirate
C. Cementation of a Modern liner into the existing socket
D. Cemented femoral stem revision
Answer: A
Reference:
https://ageinplaceschool.com/what-are-red-flags-after-hip-replacement-identifying-
complications-post-hip-replacement-surgery/
81. Figures 1 through 7 are the radiographs, MR images, and biopsy specimen of a 28-year-
old woman who is being evaluated for a left humerus lesion. She has experienced
intermittent left arm pain for 2 months and first noticed an enlarging mass 1 month ago.
Fluorescent in situ hybridization (FISH) was positive for the EWSR1 rearrangement.
Staging studies demonstrate localized disease. The optimal next step in treatment to
minimize the risk of developing metastatic disease and local recurrence is
Reference: https://clinicalsarcomaresearch.biomedcentral.com/articles/10.1186/s13569-016-
0060-4
https://ascopubs.org/doi/pdfdirect/10.1200/EDBK_390554
82. A 27-year-old female has had persistent pain in the anterior ankle since she sustained a
twisting injury while playing soccer 10 weeks ago. Physical examination shows pain on
palpation of the anterior ankle. Tibiofibular squeeze test as well as a stabilization tape test
are positive. Plain radiographs show no fracture. An MRI scan shows continuous fluid
signal from the mortise into the tibiofibular clear space noted on the T2 coronal sequence.
Figure 1 shows the diagnostic ankle arthroscopy. Which of the following is the most
definitive treatment for this patient's injury?
A. Microfracture of the lateral talar dome
B. Debridement of the syndesmosis.
C. Trans - syndesmotic fixation
D. Anterolateral ankle synovectomy
Answer: B
Surgical Options:
○ If conservative measures fail, surgical intervention may be necessary.
○ The most definitive treatment depends on the specific pathology identified
during arthroscopy:
■ Debridement of the Syndesmosis: Removal of scar tissue or
impinging structures.
■ Trans-Syndesmotic Fixation: Stabilization of the syndesmosis using
screws or other hardware.
■ Anterolateral Ankle Synovectomy: Removal of inflamed synovial
tissue.
■ Microfracture of the lateral talar dome may not directly address
anterior impingement.
83. Figures 1 and 2 are the weight-bearing radiographs of a 17-year-old girl who has great toe
pain with push-off and stiffness 1 year after undergoing a proximal crescentic osteotomy
for hallux valgus. Motion at the first metatarsophalangeal joint includes approximately
20° of dorsiflexion. What is the most appropriate treatment?
A. Proximal phalanx osteotomy.
B. Plantar flexion metatarsal osteotomy
C. Distal biplanar metatarsal osteotomy.
D. Capsular release and aggressive physical therapy.
Answer: C
Considering the patient’s age, symptoms, and the need for correction, I recommend a
combination of surgical approaches. The most appropriate treatment would likely involve
a distal biplanar metatarsal osteotomy to address both the frontal and sagittal plane
deformities. Additionally, postoperative physical therapy is essential for optimal
recovery.
Reference: https://www.orthobullets.com/foot-and-ankle/7008/hallux-valgus
84. Figure 1 is the radiograph of a 45-year-old heavy laborer who sustained an injury after he
fell from a height of two stories. According to the research, which of the following factors
is most likely to decrease the risk of surgical complications in this patient?
A. Surgeon Experience
B. Use of titanium implants
C. Use of bone graft or bone void filler
D. Allgöwer-donati suture technique
Answer: A
Surgeon Experience is a critical factor that can significantly impact the risk of surgical
complications in patients undergoing elective inpatient surgical procedures. Let’s explore
this further:
1. Association with Falls and Surgical Complications:
○ Falls after elective inpatient surgical procedures are common and can have
physical, emotional, and financial consequences1.
○ Close interactions between patients and healthcare teams before and after surgery
provide opportunities to address modifiable risk factors associated with falls.
2. Multicomponent Fall Prevention Interventions:
○ Effective fall prevention requires multicomponent interventions because fall risk
is associated with various factors.
○ While some risk factors (such as age and previous falls) are not modifiable, others
can be addressed.
○ A study investigated a perioperative multicomponent fall prevention intervention
that incorporated patient education, home medication review, and hazard
identification in the home environment1.
○ However, this specific intervention did not show a significant reduction in falls
during the first year after elective inpatient surgical procedures.
3. Surgeon Experience Matters:
○ Surgeon experience plays a crucial role in minimizing surgical complications.
○ Experienced surgeons are more adept at handling unexpected situations, making
precise incisions, and ensuring optimal outcomes.
○ Their expertise contributes to better patient safety and reduced postoperative
complications.
In summary, while various factors influence surgical outcomes, surgeon experience remains a
key determinant in decreasing the risk of complications
Reference: https://journals.plos.org/plosone/article?id=10.1371/journal.pone.0288320
85. During a minimally invasive surgery (MIS) to perform a lateral lumbar interbody fusion,
approach to which level carries the highest risk of injury to the lumbar trunk?
A. L1-L2.
B. L2- L3
C. L3- L4
D. L4-L5
Answer: D
Referensi:
Lateral Lumbar Interbody Fusion—Outcomes and Complications - PMC (nih.gov)
Lateral Lumbar Interbody Fusion: Review of Surgical Technique and Postoperative
Multimodality Imaging Findings | AJR (ajronline.org)
86. The use of demineralized bone matrix (DBM) for posterolateral lumbar fusion hasbeen
shown to
A. Have equivalent amounts of bone morphogenetic proteins (BMP) in all DBM
preparations.
B. Be an effective replacement for local bone.
C. Have similar fusion rates to Iliac Crest Bone Graft (ICBG) when combined with
Local
D. Produce higher rates of fusion compared with biphasic calcium phosphate.
Answer: C
Referensi: Demineralized Bone Matrix in Spine Surgery: A Review o Current
Applications and Future Trends untitled (ijssurgery.com)
87. A 42-year-old woman has a 3-week history of acute lower back pain with radiation into
the left lower extremity. There is no history of trauma and no systemic symptoms are
noted. Examination reveals a positive straight leg test at 25 degrees on the left side. Motor
testing reveals mild weakness of the gluteus maximus and weakness of the gastrocnemius
at 3/5. Sensory examination reveals decreased sensation along the lateral aspect of the
foot. Knee reflex is intact; however, the ankle reflex is absent. MRI scans show a
posterolateral disk herniation. The diagnosis at this time is consistent with a herniated
nucleus pulposus at what level?
A. L2-3
B. L3-4
C. L4-5
D. L5-S1
Answer: D
- Ankle Reflex:
Referensi:
Achilles Reflex - StatPearls - NCBI Bookshelf (nih.gov)
Anatomy, Abdomen and Pelvis: Superior Gluteal Nerve - StatPearls - NCBI Bookshelf
(nih.gov)
88. A 23-year-old man is evaluated in the emergency department after a diving accident.
Radiographs reveal bilateral jumped facets at C6-7. Examination reveals no motor
function below the C7 level. There is some maintained sensation in the lower extremities.
What is the patients current grade on the ASIA (American Spinal Injury Association)
impairment scale?
A. ASIA A.
B. ASIA B
C. ASIA C.
D. ASIA D.
Answer: B
No motor function below the level of injury, sensosry (+) → ASIA B
Reference: Spinal Cord Injury – Types of Injury, Diagnosis and Treatment (aans.org)
89. A 28-year-old Hispanic male assembly line worker sustains an injury while lifting a 40-
Ibs bag onto a palette. He experiences immediate low back pain, and within 5 days, he
develops severe left leg pain. His MRI scans are shown in Figures 1 and 2.
What factor is the most predictive in proceeding with surgery?
A. ethnicity
B. age
C. physician Specialty
D. gender
E. Opsi 5
Answer: D
Reference: Lumbar Disk Herniation Surgery: Outcome and Predictors - PMC (nih.gov)
90. Advanced imaging, to include MRI and CT, have been obtained in the workup of patients
with low back pain. What imaging finding has been associated with reasons for back
pain?
A. Disk degeneration
B. Facet arthropathy
C. Spinal Stenosis
D. Spondylolysis
Answer: A
Sumber:
Predictors of long-term pain and disability in patients with low back pain investigated by
magnetic resonance imaging: A longitudinal study | BMC Musculoskeletal Disorders | Full
Text (biomedcentral.com)
Magnetic Resonance Imaging and Low Back Pain in Adults: A Di... : Spine (lww.com)
91. Figure 1 is the CT scan of a 36-year-old man who fell from a roof. Eight hours later at the
emergency department he describes low-back pain with numbness and weakness in his
bilateral lower extremity. A neurologic examination reveals 2/5 strength in his quadriceps
and iliopsoas bilaterally, 2/5 strength in his right anterior tibialis and gastrocsoleus, and
1/5 strength in his left anterior tibialis and gastrocsoleus. Two hours later, strength in his
lower extremities has diminished markedly. What is the best next step?
A. Intravenous (IV) methylprednisolone with a 30-mg/kg loading dose followed by
continuous infusion of 5.4 mg/kg/hour for 24 hours
B. Immediate awake traction reduction
C. Emergent open reduction/decompression.
D. Admission to the intensive care unit for fluid resuscitation followed by
reduction/decompression when stable
Answer: C
Given the clinical scenario and findings, this patient likely has a spinal cord injury due to
trauma. The combination of low-back pain, numbness, and progressive weakness in the
bilateral lower extremities raises concern for spinal cord compression. Let’s evaluate the
options:
Therefore, the best next step for this patient is emergent open reduction/decompression
(Option D). Prompt surgical intervention is crucial to prevent further neurological
deterioration.
Reference: https://www.barrowneuro.org/for-physicians-researchers/education/grand-
rounds-publications-media/barrow-quarterly/volume-19-no-3-2003/emergent-treatment-of-
acute-spinal-cord-injuries-current-status/
92. Figure 1 and 2 show the radiographs of a 42-year-old carpenter who is evaluated for pain
in his right index finger proximal interphalangeal (PIP) joint. He has tried anti-
inflammatories as well as a steroid injection without satisfaction. He requests surgery to
alleviate the pain. What surgical option has been shown to provide the most reliable and
durable outcomes?
A. Pyrocarbon replacement.
B. Silicone replacement.
C. Resection arthroplasty.
D. P I P arthrodesis
Answer: B
Given the patient’s history of pain in the right index finger proximal interphalangeal (PIP)
joint and the desire for surgery, the most reliable and durable surgical option for PIP joint
arthritis is PIP joint arthroplasty. Let’s explore this procedure in more detail:
● PIP Joint Arthroplasty:
○ PIP joint arthroplasty involves replacing a damaged PIP joint with an artificial
implant to relieve pain and restore movement.
○ The PIP joint is in the middle of the finger and is responsible for bending and
extending.
○ Common indications for PIP joint arthroplasty include osteoarthritis and joint
deformity.
○ The goal is to restore functional range of motion and provide long-term pain
relief.
● Types of PIP Joint Replacements:
○ Several implant materials are available:
1. Silicone: Silicone implants are the most popular choice. They tend to have
the best results and typically last longer than 10 years. While they reduce pain
substantially, they may not significantly improve range of motion.
2. Metal: Metal implants (such as cobalt-chromium and metal-backed
polyethylene-titanium) have shown less range of motion and shorter
longevity before requiring replacement.
3. Pyrocarbon: Pyrocarbon implants are made from high-strength graphite and
can withstand strain without breaking. They have relative success with pain
improvement but may not restore full range of motion.
4. Ceramic: Ceramic implants, coated with a mineral, aim to prevent loosening.
Their long-term reliability is still being evaluated.
In summary, silicone implants are the most common and tend to provide reliable and
durable outcomes for PIP joint arthroplasty. However, the choice of implant should be
based on individual factors and the surgeon’s assessment.
Reference: Proximal Interphalangeal (PIP) Joint Arthroplasty (healthline.com)
93. What is the approximate ratio of the scaphoid distal (volar) blood supply to the proximal
(dorsal) blood supply?
A. 20:80.
B. 40 : 60.
C. 60: 40.
D. 70:30
Answer: A
Reference: https://www.orthobullets.com/hand/6034/scaphoid-fracture
94. Figures 1 shows a 20-years-old right-hand dominant man with a 6-month history of left
wrist pain and popping that has failed nonsurgical measures. No other positive findings
upon examination are noted. What is the most appropriate course of treatment?
A. Lunotriquetral fusion
B. Distal radioulnar joint (DRU) tenodesis
C. Triangular fibrocartilage complex (TFCC) repair
D. Extensor Carpi Ulnaris (ECU) tendon sheath reconstruction
Answer: C
Wrist popping and cracking, also known as crepitus, can be an uncomfortable sensation
that arises during daily activities. When considering treatment options for this patient
with left wrist pain and popping, let’s explore the possibilities:
Given the lack of positive findings on examination, the most appropriate course of
treatment would be to focus on addressing the specific pathology. Therefore, Triangular
fibrocartilage complex (TFCC) repair (Option C) may be the best choice
95. Figure 1 is the clinical photograph of a very functional 17years-old boy with cerebral
palsy and quadriplegia. He has no active supination but has full passive supination. His
ability to determine position and sensibility without visual input are good. Radiographs
show no osseous malalignment. Which treatment can best improve this patient's function?
A. Physical therapy for supination strengthening
B. Pronator Teres Muscle transfer
C. Humeral derotation osteotomy
D. Pronator quadratus muscle release
Answer: C
Given the clinical presentation of a 17-year-old boy with cerebral palsy and quadriplegia,
let’s explore the treatment options to improve his function:
Considering the patient’s lack of active supination, the most appropriate choice would be
humeral derotation osteotomy (Option C). This procedure can address rotational
deformities and potentially improve overall function.
96. In addition to the anterior oblique ligament, which other structure is an important
stabilizer of the thumb carpometacarpal (CMC) joint when the thumb is in a position of
opposition?
A. Osseous architecture.
B. Abductor pollicis brevis
C. Radio volar joint capsule
D. Dorsoradial ligament.
Answer: D
Reference: https://www.jospt.org/doi/pdf/10.2519/jospt.2003.33.7.386#:~:text=The
%20anterior%20oblique%2C%20radial%20collateral,and%20opposition%20of%20the
%20thumb.
97. A 35-years-old man sustained a traumatic low ulnar nerve palsy 18 months ago.
The extent of the clawing and intrinsic atrophy as well as the active radial deviation are
seen in Figures 1 through 3. No hyper extensibility of any of the proximal interphalangeal
(PIP) joints is observed. Preoperatively, the patient is not able to fully extend the PIP
joints with the wrist in neutral position and the examiner holding the
metacarpophalangeal (MCP) joints flexed. Figure 4 shows the intraoperative photograph
obtained during the intrinsic reconstruction procedure that is performed. The tendon
grafts were inserted distally into the
A. Proximal phalanx.
B. Radial lateral bands
C. First annular pulley.
D. Second annular pulley.
Answer: B
Treatment Options:
○ Operative Interventions are considered when the deformity affects the patient’s
quality of life.
■ Contracture Release and Passive Tenodesis: This procedure aims to
prevent MCP joint hyperextension.
■ Active Tendon Transfer: Tendon transfers can restore function by
rerouting tendons to compensate for intrinsic muscle weakness.
■ Radial Lateral Bands: These are commonly used for tendon
transfers in claw hand correction.
■ First Annular Pulley: Another option involves transferring
tendons through the first annular pulley.
■ Second Annular Pulley: This pulley can also be used for
tendon graft insertion.
Intraoperative Photograph (Figure 4):
○ The tendon grafts were inserted distally into one of the structures mentioned above.
○ The specific choice depends on the surgeon’s preference and the patient’s individual
anatomy.
In summary, the most definitive treatment for this patient’s claw hand deformity involves
operative intervention, such as contracture release and tendon transfer. The goal is to
restore hand function and prevent MCP joint hyperextension.
Reference: https://www.orthobullets.com/hand/6010/intrinsic-minus-hand-claw-hand
Answer: D
Reference:srep35493.pdf (nature.com)
99. What is the most common complication after distal biceps tendon repair at the elbow?
A. Lateral antebrachial cutaneous neuritis
B. Radial sensory neuritis.
C. Symptomatic heterotopic ossification.
D. Rupture of the repair.
Answer: A
Reference: Amarasooriya M, Bain GI, Roper T, Bryant K, Iqbal K, Phadnis J.
Complications After Distal Biceps Tendon Repair: A Systematic Review. Am J Sports
Med. 2020 Oct;48(12):3103-3111. doi: 10.1177/0363546519899933. Epub 2020 Feb 24.
PMID: 32091914.
100. Figures 1 and 2 are the clinical photograph and radiograph of a 27-year-old female with
medial deviation of her second toe. She says she took a misstep with this foot 4 months
ago that led to bruising of her plantar forefoot, but this subsequently resolved. Physical
examination of the second toe shows a positive drawer test. Which of the following
structures has most likely been damaged?
A. Plantar plate.
B. Medial collateral ligament
C. Extensor digitorum brevis
D. Flexor digitorum longus
Answer: A
● Pathology:
○ Plantar Plate Tear: Crossover toe can occur if there is a unilateral or partial
tear of the plantar plate, which is a broad, thick ligamentous structure spanning
the plantar aspect of the MTP joint.
○ Collateral Ligament Deficiency: A deficiency in one of the collateral ligaments
(usually the lateral collateral ligament) may contribute to the instability.
Reference : Cross over toe deformity | Radiology Reference Article | Radiopaedia.org